0 Daumen
208 Aufrufe

Aufgabe:

Es sei V = Span((1, 1, 1)^T ,(−1, 1, 1)^T). Bestimmen Sie eine Orthogonalbasis von V ,
und berechnen Sie damit die Projektion PV (x) für x = (1, 2, 0)^T und x = (0, −1, 4)^T

Avatar von

2 Antworten

+1 Daumen
 
Beste Antwort

Aloha :)

Wir haben die beiden Vektoren aus dem Erzeugendensystem:$$\vec v_1=\begin{pmatrix}1\\1\\1\end{pmatrix}\quad;\quad\vec v_2=\begin{pmatrix}-1\\1\\1\end{pmatrix}$$

Zur Orthogonalisierung projezieren wir \(\vec v_2\) auf \(\vec v_1\):$$\vec v_{2\parallel}=\frac{\vec v_1\cdot\vec v_2}{\|\vec v_1\|^2}\cdot\vec v_1=\frac{-1+1+1}{3}\begin{pmatrix}1\\1\\1\end{pmatrix}=\begin{pmatrix}\frac13\\[1ex]\frac13\\[1ex]\frac13\end{pmatrix}$$und subtrahieren den zu \(\vec v_1\) parallelen Anteil von \(\vec v_2\):$$\vec v_{2\perp}=\vec v_2-\vec v_{2\parallel}=\begin{pmatrix}-1\\1\\1\end{pmatrix}-\begin{pmatrix}\frac13\\[1ex]\frac13\\[1ex]\frac13\end{pmatrix}=\begin{pmatrix}-\frac43\\[1ex]\frac23\\[1ex]\frac23\end{pmatrix}=\frac23\begin{pmatrix}-2\\1\\1\end{pmatrix}$$

Eine mögliche Orthogonalbasis von \(V\) ist also:$$\text{Basis}_\perp(V)=\left(\begin{pmatrix}1\\1\\1\end{pmatrix},\begin{pmatrix}-2\\1\\1\end{pmatrix}\right)$$

Die Projektion von \(\vec x=(1;2;0)^T\) und \(\vec y=(0;-1;4)^T\) auf \(V\) erhalten wir durch Multiplilkation mit den normierten orthogonalen Basisvektoren:$$\vec x_P=\frac{1}{3}\underbrace{\left[\begin{pmatrix}1\\2\\0\end{pmatrix}\begin{pmatrix}1\\1\\1\end{pmatrix}\right]}_{=3}\begin{pmatrix}1\\1\\1\end{pmatrix}+\frac{1}{6}\underbrace{\left[\begin{pmatrix}1\\2\\0\end{pmatrix}\begin{pmatrix}-2\\1\\1\end{pmatrix}\right]}_{=0}\begin{pmatrix}-2\\1\\1\end{pmatrix}=\begin{pmatrix}1\\1\\1\end{pmatrix}$$$$\vec y_P=\frac{1}{3}\underbrace{\left[\begin{pmatrix}0\\-1\\4\end{pmatrix}\begin{pmatrix}1\\1\\1\end{pmatrix}\right]}_{=3}\begin{pmatrix}1\\1\\1\end{pmatrix}+\frac{1}{6}\underbrace{\left[\begin{pmatrix}0\\-1\\4\end{pmatrix}\begin{pmatrix}-2\\1\\1\end{pmatrix}\right]}_{=3}\begin{pmatrix}-2\\1\\1\end{pmatrix}=\begin{pmatrix}1\\1\\1\end{pmatrix}+\frac{1}{2}\begin{pmatrix}-2\\1\\1\end{pmatrix}=\begin{pmatrix}0\\\frac32\\[1ex]\frac32\end{pmatrix}$$

Avatar von 148 k 🚀
0 Daumen

Hallo

du kennst doch sicher Gram Schmidt? bestimme damit die Orthogonalbasis aus 2 Vektoren, der dritte  dazu senkrechte ist dann der zu R^3 fehlende , damit findest du die Projektion.

lul

Avatar von 106 k 🚀

Ein anderes Problem?

Stell deine Frage

Willkommen bei der Mathelounge! Stell deine Frage einfach und kostenlos

x
Made by a lovely community